Mathcenter Forum  

Go Back   Mathcenter Forum > คณิตศาสตร์โอลิมปิก และอุดมศึกษา > อสมการ
สมัครสมาชิก คู่มือการใช้ รายชื่อสมาชิก ปฏิทิน ข้อความวันนี้

ตั้งหัวข้อใหม่ Reply
 
เครื่องมือของหัวข้อ ค้นหาในหัวข้อนี้
  #1  
Old 07 กันยายน 2011, 19:17
catengland's Avatar
catengland catengland ไม่อยู่ในระบบ
ลมปราณบริสุทธิ์
 
วันที่สมัครสมาชิก: 14 กุมภาพันธ์ 2010
ข้อความ: 144
catengland is on a distinguished road
Default โจทย์ AM-GM 2 ข้อครับ

$ 1.ให้ a_1,a_2,...,a_n เป็นจำนวนจริงบวก n จำนวน และให้ b_1,b_2,...,b_n เป็นวิธีเรียงสับเปลี่ยนชุดหนึ่งของ a_1,a_2,...,a_n จงแสดงว่า \frac{a_1}{b_1}+\frac{a_2}{b_2}+...+\frac{a_n}{b_n}\geqslant n $
$2.สำหรับจำนวนจริงบวก a,b,c ใดๆ จงพิสูจน์ว่า
\frac{1}{a}+\frac{1}{b}+\frac{1}{c}\geqslant \frac{2}{a+b}+\frac{2}{b+c}+\frac{2}{c+a}\geqslant \frac{9}{a+b+c}$
ช่วยทีครับ
__________________
ตอบพร้อมอ้างอิงข้อความนี้
  #2  
Old 07 กันยายน 2011, 19:25
Cachy-Schwarz's Avatar
Cachy-Schwarz Cachy-Schwarz ไม่อยู่ในระบบ
บัณฑิตฟ้า
 
วันที่สมัครสมาชิก: 08 ธันวาคม 2010
ข้อความ: 404
Cachy-Schwarz is on a distinguished road
Default

อ้างอิง:
ข้อความเดิมเขียนโดยคุณ catengland View Post
$ 1.ให้ a_1,a_2,...,a_n เป็นจำนวนจริงบวก n จำนวน และให้ b_1,b_2,...,b_n เป็นวิธีเรียงสับเปลี่ยนชุดหนึ่งของ a_1,a_2,...,a_n จงแสดงว่า \frac{a_1}{b_1}+\frac{a_2}{b_2}+...+\frac{a_n}{b_n}\geqslant n $
$2.สำหรับจำนวนจริงบวก a,b,c ใดๆ จงพิสูจน์ว่า
\frac{1}{a}+\frac{1}{b}+\frac{1}{c}\geqslant \frac{2}{a+b}+\frac{2}{b+c}+\frac{2}{c+a}\geqslant \frac{9}{a+b+c}$
ช่วยทีครับ
ข้อสองใช้ AM-HM ครับ ช่วงเเรกลองจับทีละ 2 ตัวครับ
$\frac{1}{a} +\frac{1}{b}\geqslant \frac{4}{a+b}$
ตอบพร้อมอ้างอิงข้อความนี้
  #3  
Old 07 กันยายน 2011, 19:28
จูกัดเหลียง's Avatar
จูกัดเหลียง จูกัดเหลียง ไม่อยู่ในระบบ
ลมปราณไร้สภาพ
 
วันที่สมัครสมาชิก: 21 กุมภาพันธ์ 2011
ข้อความ: 1,234
จูกัดเหลียง is on a distinguished road
Default

อ้างอิง:
ข้อความเดิมเขียนโดยคุณ catengland View Post
2.สำหรับจำนวนจริงบวก $a,b,c$ ใดๆ จงพิสูจน์ว่า
$$ \frac{1}{a}+\frac{1}{b}+\frac{1}{c}\geqslant \frac{2}{a+b}+\frac{2}{b+c}+\frac{2}{c+a}\geqslant \frac{9}{a+b+c}$$
ช่วยทีครับ
โดย A.M.-G.M. $\frac{1}{a}+\frac{1}{b}\ge 2\sqrt{\frac{1}{ab}}\ge \frac{4}{a+b}$ ทำนองเดียวกันก็คิดต่อไปเเล้วนำมาบวกกัน
ทำให้ได้ว่า $$\frac{1}{a}+\frac{1}{b}+\frac{1}{c}\ge \frac{2}{a+b}+\frac{2}{b+c}+\frac{2}{c+a}$$
เเละจากอสมการ Cauchy $$\frac{1}{a}+\frac{1}{b}+\frac{1}{c}\ge \frac{2}{a+b}+\frac{2}{b+c}+\frac{2}{c+a}$$ $$= \frac{1}{\frac{1}{2}(a+b)}+\frac{1}{\frac{1}{2}(b+c)}+\frac{1}{\frac{1}{2}(c+a)} \ge \frac{(1+1+1)^2}{a+b+c}=\frac{9}{a+b+c}$$
__________________
Vouloir c'est pouvoir
ตอบพร้อมอ้างอิงข้อความนี้
  #4  
Old 07 กันยายน 2011, 22:17
PP_nine's Avatar
PP_nine PP_nine ไม่อยู่ในระบบ
กระบี่ประสานใจ
 
วันที่สมัครสมาชิก: 24 เมษายน 2010
ข้อความ: 607
PP_nine is on a distinguished road
Default

ข้อแรกใช้ AM-GM ธรรมดานี่ครับ

(ใส่ AM-GM ธรรมดาแล้วข้างในตัวรากที่ n จะตัดกันหมดเป็น 1)
__________________
keep your way.
ตอบพร้อมอ้างอิงข้อความนี้
  #5  
Old 08 กันยายน 2011, 13:22
กิตติ's Avatar
กิตติ กิตติ ไม่อยู่ในระบบ
กระบี่ธรรมชาติ
 
วันที่สมัครสมาชิก: 08 พฤศจิกายน 2009
ข้อความ: 2,723
กิตติ is on a distinguished road
Default

ข้อ2....ตอนพิสูจน์ว่า
$\frac{2}{a+b} +\frac{2}{b+c} +\frac{2}{a+c} \geqslant \frac{9}{a+b+c} $ ใช้ $AM-GM-HM$ ก็ได้

$\frac{\frac{1}{a+b} +\frac{1}{b+c} +\frac{1}{a+c}}{3} \geqslant \frac{3}{(a+b)+(b+c)+(a+c)} $

$\frac{1}{a+b} +\frac{1}{b+c} +\frac{1}{a+c} \geqslant \frac{9}{2(a+b+c)} $

$\frac{2}{a+b} +\frac{2}{b+c} +\frac{2}{a+c} \geqslant \frac{9}{(a+b+c)}$
__________________
"ถ้าเราล้มบ่อยๆ ในที่สุดเราจะรู้ว่าถ้าจะล้ม ล้มท่าไหนจะเจ็บน้อยที่สุด และรู้อีกว่าต่อไปทำยังไงจะไม่ให้ล้มอีก
ดังนั้นจงอย่ากลัวที่จะล้ม
"...อาจารย์อำนวย ขนันไทย
ครั้งแรกในชีวิตที่สอบคณิตสมาคมคณิตศาสตร์เมื่อปี2533...ผมได้แค่24คะแนน(จากร้อยคะแนน)
ตอบพร้อมอ้างอิงข้อความนี้
  #6  
Old 11 กันยายน 2011, 13:52
จูกัดเหลียง's Avatar
จูกัดเหลียง จูกัดเหลียง ไม่อยู่ในระบบ
ลมปราณไร้สภาพ
 
วันที่สมัครสมาชิก: 21 กุมภาพันธ์ 2011
ข้อความ: 1,234
จูกัดเหลียง is on a distinguished road
Default

อ้างอิง:
ข้อความเดิมเขียนโดยคุณ PP_nine View Post
ข้อแรกใช้ AM-GM ธรรมดานี่ครับ

(ใส่ AM-GM ธรรมดาแล้วข้างในตัวรากที่ n จะตัดกันหมดเป็น 1)
ยังไงครับ ช่วยเเสดงวิธีให้ดูหน่อย
__________________
Vouloir c'est pouvoir
ตอบพร้อมอ้างอิงข้อความนี้
  #7  
Old 11 กันยายน 2011, 17:38
catengland's Avatar
catengland catengland ไม่อยู่ในระบบ
ลมปราณบริสุทธิ์
 
วันที่สมัครสมาชิก: 14 กุมภาพันธ์ 2010
ข้อความ: 144
catengland is on a distinguished road
Default

คือข้อ 1 ผมไม่เข้าใจคำว่า $ให้ b_1,b_2,...,b_n เป็นวิธีเรียงสับเปลี่ยนชุดหนึ่งของ a_1,a_2,...,a_n $ อะครับช่วยอธิบายให้ทีครับ
__________________
ตอบพร้อมอ้างอิงข้อความนี้
  #8  
Old 11 กันยายน 2011, 17:52
Onasdi's Avatar
Onasdi Onasdi ไม่อยู่ในระบบ
กระบี่ประสานใจ
 
วันที่สมัครสมาชิก: 12 พฤษภาคม 2005
ข้อความ: 760
Onasdi is on a distinguished road
Default

ก็อย่างเช่นว่า ถ้า $n=4,\,\, a:1,3,3,8$ แล้ว $b$ ก็อาจจจะเป็น $1,8,3,3$ หรือ $3,8,1,3$ และอื่นๆ
ตอบพร้อมอ้างอิงข้อความนี้
  #9  
Old 14 กันยายน 2011, 18:28
PP_nine's Avatar
PP_nine PP_nine ไม่อยู่ในระบบ
กระบี่ประสานใจ
 
วันที่สมัครสมาชิก: 24 เมษายน 2010
ข้อความ: 607
PP_nine is on a distinguished road
Default

อ้างอิง:
ข้อความเดิมเขียนโดยคุณ catengland View Post
1.ให้ $a_1,a_2,...,a_n$ เป็นจำนวนจริงบวก $n$ จำนวน และให้ $b_1,b_2,...,b_n$ เป็นวิธีเรียงสับเปลี่ยนชุดหนึ่งของ $a_1,a_2,...,a_n$ จงแสดงว่า $\frac{a_1}{b_1}+\frac{a_2}{b_2}+...+\frac{a_n}{b_n}\geqslant n $
$$\frac{a_1}{b_1}+\frac{a_2}{b_2}+...+\frac{a_n}{b_n}\geqslant n \sqrt[n]{\frac{a_n}{b_n} \cdot \frac{a_n}{b_n} \cdot ...\frac{a_n}{b_n}}=n$$
เพราะลำดับ $b_n$ เป็นการเรียงสับเปลี่ยนของลำดับ $a_n$ จึงได้ว่าทุกตัวต้องมีคู่ของมันมาหารกันเสมอ เลยได้ 1
__________________
keep your way.
ตอบพร้อมอ้างอิงข้อความนี้
  #10  
Old 16 กันยายน 2011, 20:22
catengland's Avatar
catengland catengland ไม่อยู่ในระบบ
ลมปราณบริสุทธิ์
 
วันที่สมัครสมาชิก: 14 กุมภาพันธ์ 2010
ข้อความ: 144
catengland is on a distinguished road
Default

ผมขอถามโจทย์เพิ่มนะครับ
จงพิสูจน์ว่า
1. จงพิสูจน์ว่า $(n+1)^{\frac{n-1}{2}}$ $<$ $n!$ $<$ $n^n$ และ ให้ $n$ เป็นจำนวนนับโดยที่ $n\geqslant 2$

2.จงแสดงว่า $\frac{1}{n+1}+\frac{1}{n+2}+\frac{1}{n+3}+...+\frac{1}{2n}$ $\geqslant \frac{1}{2}$ สำหรับจำนวนนับ $n$ ใด ๆ

3.จงแสดงว่า $(n!)^2 \geqslant (n+1)^{n-1}$ สำหรับจำนวนนับ $n$ ใด ๆ

4.ในรูปสามเหลี่ยมใด ๆ จงพิสูจน์ว่า 32 เท่าของพื้นที่มีค่าไม่เกินกำลังสองของความยาวรอบรูป
__________________
ตอบพร้อมอ้างอิงข้อความนี้
  #11  
Old 16 กันยายน 2011, 20:44
Amankris's Avatar
Amankris Amankris ไม่อยู่ในระบบ
กระบี่ธรรมชาติ
 
วันที่สมัครสมาชิก: 13 มกราคม 2007
ข้อความ: 2,492
Amankris is on a distinguished road
Default

1,2,3
เป็นแบบฝึกหัดเรื่องอุปนัย

4
Heron ก็น่าจะหลุดนะ
Edited : ไม่จริงนี่ครับ

16 กันยายน 2011 22:00 : ข้อความนี้ถูกแก้ไขแล้ว 1 ครั้ง, ครั้งล่าสุดโดยคุณ Amankris
ตอบพร้อมอ้างอิงข้อความนี้
  #12  
Old 16 กันยายน 2011, 21:20
จูกัดเหลียง's Avatar
จูกัดเหลียง จูกัดเหลียง ไม่อยู่ในระบบ
ลมปราณไร้สภาพ
 
วันที่สมัครสมาชิก: 21 กุมภาพันธ์ 2011
ข้อความ: 1,234
จูกัดเหลียง is on a distinguished road
Default

อ้างอิง:
ข้อความเดิมเขียนโดยคุณ catengland View Post

2.จงแสดงว่า $\frac{1}{n+1}+\frac{1}{n+2}+\frac{1}{n+3}+...+\frac{1}{2n}$ $\geqslant \frac{1}{2}$ สำหรับจำนวนนับ $n$ ใด ๆ
ขอข้อสองละกันนะครับ
โดย อสมการ A.M.-H.M.
$$\Big(\frac{1}{n+1}+\frac{1}{n+2}+\frac{1}{n+3}+...+\frac{1}{2n}\Big)((n+1)+(n+2)+(n+3)+...+2n)\ge n^2$$
$$\rightarrow \Big(\frac{1}{n+1}+\frac{1}{n+2}+\frac{1}{n+3}+...+\frac{1}{2n}\Big)\ge \frac{n^2}{(n+1)+(n+2)+(n+3)+...+2n}=\frac{2n}{3n+1}\ge \frac{1}{2}$$
__________________
Vouloir c'est pouvoir
ตอบพร้อมอ้างอิงข้อความนี้
  #13  
Old 16 กันยายน 2011, 21:49
จูกัดเหลียง's Avatar
จูกัดเหลียง จูกัดเหลียง ไม่อยู่ในระบบ
ลมปราณไร้สภาพ
 
วันที่สมัครสมาชิก: 21 กุมภาพันธ์ 2011
ข้อความ: 1,234
จูกัดเหลียง is on a distinguished road
Default

อ้างอิง:
ข้อความเดิมเขียนโดยคุณ catengland View Post

4.ในรูปสามเหลี่ยมใด ๆ จงพิสูจน์ว่า 32 เท่าของพื้นที่มีค่าไม่เกินกำลังสองของความยาวรอบรูป
ข้อนี่ผมว่ามันเเปลกๆอ่ะครับ
จากโจทย์ ให้ความยาวด้านของ 3 เหลี่ยมเป็น $a,b,c$
เราจึงจะพิสูจน์ว่า $$32\sqrt{\Big(\frac{a+b+c}{2}\Big)\Big(\frac{a-b+c}{2}\Big)\Big(\frac{a+b-c}{2}\Big)\Big(\frac{-a+b+c}{2}\Big)}\leq (a+b+c)^2\leftrightarrow (a+b+c)^2\ge 8\sqrt{(a+b+c)(a-b+c)(b+c-a)(c+b-a)}$$
เเต่จาก A.M.-G.M. $$\Big((a-b+c)+(b-c+a)+(c-a+b)\Big)\ge 3\sqrt[3]{(a-b+c)(b+c-a)(c+b-a)}\rightarrow (a+b+c)^{\frac{3}{2}}\ge 3\sqrt{3}\sqrt{(a-b+c)(b+c-a)(c+b-a)}$$
$$\rightarrow (a+b+c)^2\ge 3\sqrt{3}\sqrt{(a+b+c)(a-b+c)(b+c-a)(c+b-a)}$$
ซึ่งมั่นเเปลกๆอ่ะครับ
__________________
Vouloir c'est pouvoir
ตอบพร้อมอ้างอิงข้อความนี้
  #14  
Old 19 กันยายน 2011, 01:31
Keehlzver's Avatar
Keehlzver Keehlzver ไม่อยู่ในระบบ
บัณฑิตฟ้า
 
วันที่สมัครสมาชิก: 26 มกราคม 2009
ข้อความ: 533
Keehlzver is on a distinguished road
Default

รบกวนคุณ Amankris ช่วย Contradict ให้ดูหน่อยครับ เพราะผมลองแล้วอสมการมันจริงครับ

(ผมลอง Store ตัวแปรไม่กี่ค่าครับ)
__________________
"ชั่วโมงหน้าต้องดีกว่าเดิม!"
ตอบพร้อมอ้างอิงข้อความนี้
  #15  
Old 19 กันยายน 2011, 03:06
Amankris's Avatar
Amankris Amankris ไม่อยู่ในระบบ
กระบี่ธรรมชาติ
 
วันที่สมัครสมาชิก: 13 มกราคม 2007
ข้อความ: 2,492
Amankris is on a distinguished road
Default

#14
ลองสามเหลี่ยมด้านเท่ายังครับ
ตอบพร้อมอ้างอิงข้อความนี้
ตั้งหัวข้อใหม่ Reply



กฎการส่งข้อความ
คุณ ไม่สามารถ ตั้งหัวข้อใหม่ได้
คุณ ไม่สามารถ ตอบหัวข้อได้
คุณ ไม่สามารถ แนบไฟล์และเอกสารได้
คุณ ไม่สามารถ แก้ไขข้อความของคุณเองได้

vB code is On
Smilies are On
[IMG] code is On
HTML code is Off
ทางลัดสู่ห้อง


เวลาที่แสดงทั้งหมด เป็นเวลาที่ประเทศไทย (GMT +7) ขณะนี้เป็นเวลา 13:48


Powered by vBulletin® Copyright ©2000 - 2024, Jelsoft Enterprises Ltd.
Modified by Jetsada Karnpracha